LSAT and Law School Admissions Forum

Get expert LSAT preparation and law school admissions advice from PowerScore Test Preparation.

 Administrator
PowerScore Staff
  • PowerScore Staff
  • Posts: 8917
  • Joined: Feb 02, 2011
|
#78478
Complete Question Explanation
The correct answer choice is (B).

Answer choice (A):

Answer choice (B): This is the correct answer choice.

Answer choice (C):

Answer choice (D):

Answer choice (E):

This explanation is still in progress. Please post any questions below!
User avatar
 itsmathuri
  • Posts: 5
  • Joined: Mar 21, 2021
|
#86985
Hi there,

Is A incorrect because it must be true?

Thanks
User avatar
 Ryan Twomey
PowerScore Staff
  • PowerScore Staff
  • Posts: 141
  • Joined: Mar 04, 2021
|
#87014
Hey itsmathuri,

If W is in the morning then R must be at 9, Z must be at 10, W must be at 11, Y must be at 2, and T and S are interchangable at 3 and 4 pm.

You could do 3 templates in this game if that helps you out with the WY block. It can only go in 11/2, 2/3, and 3/4.

So in this question, A is incorrect because it must be false.

I hope this helps. Maybe try redoing the setup again with the templates I outlined and that might help you see the game more clearly. But also, the incorrect answers in a could be true question will be because they must be fale, not because they must be true.

Good luck in your studies and I wish you all the best.

Best,
Ryan
User avatar
 Miajmw
  • Posts: 2
  • Joined: Jun 12, 2021
|
#88253
I don’t understand why C is wrong? The set up would be RZS TWY how is that wrong?
User avatar
 Miajmw
  • Posts: 2
  • Joined: Jun 12, 2021
|
#88254
Please ignore my previous post, made a careless mistake

Get the most out of your LSAT Prep Plus subscription.

Analyze and track your performance with our Testing and Analytics Package.